Những câu hỏi liên quan
Nguyễn Thu Huyền
Xem chi tiết
tthnew
18 tháng 8 2019 lúc 8:34

By Cauchy-Schwarz, we have:

\(VT\ge\frac{\left(a^3+b^3+c^3\right)^2}{2\left(a^3+b^3+c^3\right)+a^2b+b^2c+c^2a}\)

We will prove: \(a^2b+b^2c+c^2a\le a^3+b^3+c^3\)

\(\Leftrightarrow a^2b+b^2c+c^2a+3abc\le a^3+b^3+c^3+3abc\)

By Schur, we have: \(RHS\ge ab\left(a+b\right)+bc\left(b+c\right)+ca\left(a\right)\)

So we're only need to prove: \(ab\left(a+b\right)+bc\left(b+c\right)+ca\left(c+a\right)\ge a^2b+b^2c+c^2a+3abc\)

\(\Leftrightarrow ab^2+bc^2+ca^2\ge3abc\)

It is true by AM-GM ineq', so we have Q.E.D.

P/s: Em thử giải bài này bằng tiếng Anh (để tự luyện kĩ năng tiếng anh, tí em giải lại theo tiếng việt)

Bình luận (5)
tthnew
18 tháng 8 2019 lúc 8:35

Ấy nhầm:V

By Schur, we have \(RHS\ge ab\left(a+b\right)+bc\left(b+c\right)+ca\left(c+a\right)\)

So we're only need to prove \(ab\left(a+b\right)+bc\left(b+c\right)+ca\left(c+a\right)\ge a^2b+b^2c+c^2a\)

Còn lại y chang:v

Bình luận (0)
tthnew
18 tháng 8 2019 lúc 8:42

Làm màu bằng tiếng anh và cái kết...:V (nãy làm nhầm, phải sửa lại đó)

Áp dụng BĐT Cauchy-Schwarz dạng Engel,ta có:

\(VT\ge\frac{\left(a^3+b^3+c^3\right)^2}{2\left(a^3+b^3+c^3\right)+a^2b+b^2c+c^2a}\)

Ta sẽ chứng minh \(a^2b+b^2c+c^2a\le a^3+b^3+c^3\) (để từ đó suy ra đpcm)

Thật vậy, thêm 3abc vào hai vế, BĐT cần chứng minh tương đương:

\(a^3+b^3+c^3+3abc\ge a^2b+b^2c+c^2a+3abc\).

Áp dụng BĐT Schur, \(VT=a^3+b^3+c^3+3abc\ge ab\left(a+b\right)+bc\left(b+c\right)+ca\left(c+a\right)\)

Do đó ta chỉ cần chứng minh \(ab\left(a+b\right)+bc\left(b+c\right)+ca\left(c+a\right)\ge a^2b+b^2c+c^2a+3abc\)

Hay \(ab^2+bc^2+ca^2\ge3abc\). BĐT này đúng theo AM-GM

Bình luận (5)
l҉o҉n҉g҉ d҉z҉
Xem chi tiết
HD Film
25 tháng 7 2020 lúc 12:08

Câu 1:
\(4\sqrt[4]{\left(a+1\right)\left(b+4\right)\left(c-2\right)\left(d-3\right)}\le a+1+b+4+c-2+d-3=a+b+c+d\)

Dấu = xảy ra khi a = -1; b = -4; c = 2; d= 3

Bình luận (0)
 Khách vãng lai đã xóa
Phùng Minh Quân
25 tháng 7 2020 lúc 12:14

\(\frac{a^2}{b^5}+\frac{1}{a^2b}\ge\frac{2}{b^3}\)\(\Leftrightarrow\)\(\frac{a^2}{b^5}\ge\frac{2}{b^3}-\frac{1}{a^2b}\)

\(\frac{2}{a^3}+\frac{1}{b^3}\ge\frac{3}{a^2b}\)\(\Leftrightarrow\)\(\frac{1}{a^2b}\le\frac{2}{3a^3}+\frac{1}{3b^3}\)

\(\Rightarrow\)\(\Sigma\frac{a^2}{b^5}\ge\Sigma\left(\frac{5}{3b^3}-\frac{2}{3a^3}\right)=\frac{1}{a^3}+\frac{1}{b^3}+\frac{1}{c^3}+\frac{1}{d^3}\)

Bình luận (0)
 Khách vãng lai đã xóa
Nguyen Phan Hung Cuong
25 tháng 7 2020 lúc 19:59

ta sẽ giết ngươi kí tên dép đờ kiu lờ

Bình luận (0)
 Khách vãng lai đã xóa
Hà My Trần
Xem chi tiết
N.T.M.D
Xem chi tiết
Nguyễn Việt Lâm
6 tháng 5 2021 lúc 17:18

Ta chứng minh BĐT sau với các số dương:

\(\dfrac{1}{x}+\dfrac{1}{y}\ge\dfrac{4}{x+y}\)

Thật vậy, BĐT tương đương: \(\dfrac{x+y}{xy}\ge\dfrac{4}{x+y}\Leftrightarrow\left(x+y\right)^2\ge4xy\)

\(\Leftrightarrow x^2-2xy+y^2\ge0\Leftrightarrow\left(x-y\right)^2\ge0\) (luôn đúng)

Áp dụng:

\(\dfrac{1}{a}+\dfrac{1}{b}\ge\dfrac{4}{a+b}\) ; \(\dfrac{1}{b}+\dfrac{1}{c}\ge\dfrac{4}{b+c}\) ; \(\dfrac{1}{c}+\dfrac{1}{a}\ge\dfrac{4}{c+a}\)

Cộng vế với vế:

\(2\left(\dfrac{1}{a}+\dfrac{1}{b}+\dfrac{1}{c}\right)\ge\dfrac{4}{a+b}+\dfrac{4}{b+c}+\dfrac{4}{c+a}\)

\(\Leftrightarrow\dfrac{1}{a}+\dfrac{1}{b}+\dfrac{1}{c}\ge\dfrac{2}{a+b}+\dfrac{2}{b+c}+\dfrac{2}{c+a}\)

Bình luận (0)
Nguyễn Việt Lâm
6 tháng 5 2021 lúc 17:20

b.

Ta có:

\(\dfrac{1}{a}+\dfrac{1}{b}\ge\dfrac{4}{a+b}\Rightarrow\dfrac{3}{a}+\dfrac{3}{b}\ge\dfrac{12}{a+b}\) (1)

\(\dfrac{1}{b}+\dfrac{1}{c}\ge\dfrac{4}{b+c}\Rightarrow\dfrac{2}{b}+\dfrac{2}{c}\ge\dfrac{8}{b+c}\) (2)

\(\dfrac{1}{c}+\dfrac{1}{a}\ge\dfrac{4}{c+a}\) (3)

Cộng vế với vế (1); (2) và (3):

\(\dfrac{4}{a}+\dfrac{5}{b}+\dfrac{3}{c}\ge4\left(\dfrac{3}{a+b}+\dfrac{2}{b+c}+\dfrac{1}{c+a}\right)\) (đpcm)

Dấu "=" xảy ra khi \(a=b=c\)

Bình luận (0)
N.T.M.D
Xem chi tiết
An Vy
Xem chi tiết
ngoc bich 2
Xem chi tiết
Trần Phúc Khang
7 tháng 8 2019 lúc 10:58

Áp dụng BĐT cosi ta có

\(\frac{1}{a^3}+\frac{1}{a^3}+\frac{1}{b^3}\ge\frac{3}{a^2b}\)\(\frac{1}{b^3}+\frac{1}{b^3}+\frac{1}{c^3}\ge\frac{3}{b^2c}\)\(\frac{1}{c^3}+\frac{1}{c^3}+\frac{1}{d^3}\ge\frac{3}{c^2d}\)

\(\frac{1}{d^3}+\frac{1}{d^3}+\frac{1}{a^3}\ge\frac{3}{d^2a}\)

Cộng các BĐt trên ta có 

\(\frac{1}{a^3}+\frac{1}{b^3}+\frac{1}{c^3}+\frac{1}{d^3}\ge\frac{1}{a^2b}+\frac{1}{b^2c}+\frac{1}{c^2d}+\frac{1}{d^2a}\)(1)

Áp dụng BĐT buniacoxki ta có

\(\left(\frac{a^2}{b^5}+\frac{b^2}{c^5}+\frac{c^2}{d^5}+\frac{d^2}{a^5}\right)\left(\frac{1}{a^2b}+\frac{1}{b^2c}+\frac{1}{c^2d}+\frac{1}{d^2a}\right)\ge \left(\frac{1}{a^3}+\frac{1}{b^3}+\frac{1}{c^3}+\frac{1}{d^3}\right)^2\)

Kết hợp với (1)  ta được ĐPCM

Dấu bằng xảy ra khi a=b=c

Bình luận (0)
An Vy
Xem chi tiết
Incursion_03
2 tháng 7 2019 lúc 20:55

Áp dụng Svac + Cô-si 3 số được

\(\frac{a^5}{bc}+\frac{b^5}{ca}+\frac{c^5}{ab}=\frac{a^6}{abc}+\frac{b^6}{abc}+\frac{c^6}{abc}\ge\frac{\left(a^3+b^3+c^3\right)^2}{3abc}\ge\frac{\left(a^3+b^3+c^3\right)^2}{a^3+b^3+c^3}=VP\left(đpcm\right)\)

"=" tại a = b = c

Bình luận (0)
Tiến Nguyễn Minh
Xem chi tiết
Vũ Tiến Manh
21 tháng 10 2019 lúc 22:19

1) Áp dụng bunhiacopxki ta được \(\sqrt{\left(2a^2+b^2\right)\left(2a^2+c^2\right)}\ge\sqrt{\left(2a^2+bc\right)^2}=2a^2+bc\), tương tự với các mẫu ta được vế trái \(\le\frac{a^2}{2a^2+bc}+\frac{b^2}{2b^2+ac}+\frac{c^2}{2c^2+ab}\le1< =>\)\(1-\frac{bc}{2a^2+bc}+1-\frac{ac}{2b^2+ac}+1-\frac{ab}{2c^2+ab}\le2< =>\)

\(\frac{bc}{2a^2+bc}+\frac{ac}{2b^2+ac}+\frac{ab}{2c^2+ab}\ge1\)<=> \(\frac{b^2c^2}{2a^2bc+b^2c^2}+\frac{a^2c^2}{2b^2ac+a^2c^2}+\frac{a^2b^2}{2c^2ab+a^2b^2}\ge1\)  (1) 

áp dụng (x2 +y2 +z2)(m2+n2+p2\(\ge\left(xm+yn+zp\right)^2\)

(2a2bc +b2c2 + 2b2ac+a2c2 + 2c2ab+a2b2). VT\(\ge\left(bc+ca+ab\right)^2\)   <=> (ab+bc+ca)2. VT \(\ge\left(ab+bc+ca\right)^2< =>VT\ge1\)  ( vậy (1) đúng)

dấu '=' khi a=b=c

Bình luận (0)
 Khách vãng lai đã xóa
HD Film
21 tháng 10 2019 lúc 22:26

4b, \(\frac{a^3}{a^2+b^2}+\frac{b^3}{b^2+c^2}+\frac{c^3}{c^2+a^2}=1-\frac{ab^2}{a^2+b^2}+1-\frac{bc^2}{b^2+c^2}+1-\frac{ca^2}{a^2+c^2}\)

\(\ge3-\frac{ab^2}{2ab}-\frac{bc^2}{2bc}-\frac{ca^2}{2ac}=3-\frac{\left(a+b+c\right)}{2}=\frac{3}{2}\)

Bình luận (0)
 Khách vãng lai đã xóa
HD Film
21 tháng 10 2019 lúc 22:35

4c, 

\(\frac{a+1}{b^2+1}+\frac{b+1}{c^2+1}+\frac{c+1}{a^2+1}=a+b+c-\frac{b^2}{b^2+1}-\frac{c^2}{c^2+1}-\frac{a^2}{a^2+1}+3--\frac{b^2}{b^2+1}-\frac{c^2}{c^2+1}-\frac{a^2}{a^2+1}\)\(\ge6-2\cdot\frac{\left(a+b+c\right)}{2}=3\)

Bình luận (0)
 Khách vãng lai đã xóa